Difference between revisions of "2021 Fall AMC 12B Problems/Problem 18"
(Adding the See Also box) |
MRENTHUSIASM (talk | contribs) |
||
Line 5: | Line 5: | ||
This sequence tends to a limit; call it <math>L</math>. What is the least value of <math>k</math> such that <cmath>|u_k-L| \le \frac{1}{2^{1000}}?</cmath> | This sequence tends to a limit; call it <math>L</math>. What is the least value of <math>k</math> such that <cmath>|u_k-L| \le \frac{1}{2^{1000}}?</cmath> | ||
− | <math> | + | <math>\textbf{(A)}\: 10\qquad\textbf{(B)}\: 87\qquad\textbf{(C)}\: 123\qquad\textbf{(D)}\: 329\qquad\textbf{(E)}\: 401</math> |
==Solution== | ==Solution== | ||
− | If we list out the first few values of k, we get the series <math>\frac{1}{4}, \frac{3}{8}, \frac{15}{32}, \frac{255}{512}</math>, which seem to always be a negative power of 2 away from <math>\frac{1}{2}</math>. We can test this out by setting <math>u_k</math> to <math>\frac{1}{2}-\frac{1}{2^{n_k}}</math>. | + | If we list out the first few values of <math>k</math>, we get the series <math>\frac{1}{4}, \frac{3}{8}, \frac{15}{32}, \frac{255}{512}</math>, which seem to always be a negative power of <math>2</math> away from <math>\frac{1}{2}</math>. We can test this out by setting <math>u_k</math> to <math>\frac{1}{2}-\frac{1}{2^{n_k}}</math>. |
Now, | Now, | ||
Line 23: | Line 23: | ||
We see that <math>n_k</math> seems to always be <math>1</math> above a power of <math>2</math>. We can prove this using induction. | We see that <math>n_k</math> seems to always be <math>1</math> above a power of <math>2</math>. We can prove this using induction. | ||
− | Claim | + | <u><b>Claim</b></u> |
− | + | <math>n_k = 2^k+1</math> | |
− | Induction | + | <u><b>Base case</b></u> |
+ | |||
+ | We have <math>n_0=2^0+1</math>, which is true. | ||
+ | |||
+ | <u><b>Induction Step</b></u> | ||
+ | |||
+ | Assuming that the claim is true, we have <math>n_{k+1}=2 \cdot 2^k+2-1=2^{k+1}+1</math>. | ||
It follows that <math>n_{10}=2^{10}+1>1000</math>, and <math>n_9=2^9+1<1000</math>. Therefore, the least value of <math>k</math> would be <math>\boxed{\textbf{(A) }10}</math>. | It follows that <math>n_{10}=2^{10}+1>1000</math>, and <math>n_9=2^9+1<1000</math>. Therefore, the least value of <math>k</math> would be <math>\boxed{\textbf{(A) }10}</math>. |
Revision as of 16:53, 17 March 2022
Problem
Set , and for let be determined by the recurrence
This sequence tends to a limit; call it . What is the least value of such that
Solution
If we list out the first few values of , we get the series , which seem to always be a negative power of away from . We can test this out by setting to .
Now,
This means that this series approaches , as the second term is decreasing. In addition, we find that .
We see that seems to always be above a power of . We can prove this using induction.
Claim
Base case
We have , which is true.
Induction Step
Assuming that the claim is true, we have .
It follows that , and . Therefore, the least value of would be .
-ConcaveTriangle
See Also
2021 Fall AMC 12B (Problems • Answer Key • Resources) | |
Preceded by Problem 17 |
Followed by Problem 19 |
1 • 2 • 3 • 4 • 5 • 6 • 7 • 8 • 9 • 10 • 11 • 12 • 13 • 14 • 15 • 16 • 17 • 18 • 19 • 20 • 21 • 22 • 23 • 24 • 25 | |
All AMC 12 Problems and Solutions |
The problems on this page are copyrighted by the Mathematical Association of America's American Mathematics Competitions.